Showing that the Closure of a Connected set

  • Thread starter Thread starter trap101
  • Start date Start date
  • Tags Tags
    closure Set
Click For Summary
The discussion focuses on proving that the closure of a connected set is also connected. The initial assumption is that the closure of a connected set S is disconnected, leading to a contradiction. It is established that if S is contained in one of the disjoint subsets U or V, then S must not intersect the other subset, which implies that the closure of S cannot be disconnected. The conversation emphasizes the importance of understanding the definitions of connectedness and the properties of closures in this context. Ultimately, the proof hinges on the relationship between the subsets and the closure operator.
trap101
Messages
339
Reaction score
0
Showing that the Closure of a Connected set...

Show that the Closure of a Connected set is connected.


Attempt: Assume that the closure of a conncted set S is disconnected.

==> S = U \cup V is a disconnection of S. (bold for closure)

==> (S\capU) \cup (S\capV) is a disconnection of S.

This is where I'm stuck, I know some how I'm suppose to get a contradiction in that the closure of this set is actually connected. But I can't see how to form it.

In the solutions they make use of (S\capU) or (S\capV) being empty, but I still wasn't able to follow that either.
 
Physics news on Phys.org


A little more detail please- what do you mean by a "disconnection of S"?
 


HallsofIvy said:
A little more detail please- what do you mean by a "disconnection of S"?

oh, by the definition of connectedness or disconnectedness I guess since we don't define connectedness per say. So I mean the union of two non-empty subsets in which neither intersects the closure of the other one.
 


Ok, so you have that \overline{S}=U\cup V, which is good. But since S\subset\overline{S} and S is connected, you must have that either S\subset U or S\subset V since otherwise S would be disconnected. Let's assume without loss of generality that S\subset U.

Then in fact, S\cap V=∅ since U and V are disjoint (where your textbook's hint comes in). Moreover, we must have S\subset V^C. Since the closure operator respects subsets and V^C is closed (as V is open), we can say some pretty interesting things about \overline{S}... Remember that by assumption, \overline{S}=U\cup V.

Hope I haven't given too much away :)
 
Question: A clock's minute hand has length 4 and its hour hand has length 3. What is the distance between the tips at the moment when it is increasing most rapidly?(Putnam Exam Question) Answer: Making assumption that both the hands moves at constant angular velocities, the answer is ## \sqrt{7} .## But don't you think this assumption is somewhat doubtful and wrong?

Similar threads

  • · Replies 1 ·
Replies
1
Views
2K
  • · Replies 4 ·
Replies
4
Views
2K
  • · Replies 8 ·
Replies
8
Views
2K
  • · Replies 14 ·
Replies
14
Views
2K
Replies
2
Views
2K
Replies
6
Views
3K
Replies
11
Views
11K
  • · Replies 2 ·
Replies
2
Views
1K
  • · Replies 2 ·
Replies
2
Views
2K
  • · Replies 2 ·
Replies
2
Views
8K